Quantcast
  • Register
PhysicsOverflow is a next-generation academic platform for physicists and astronomers, including a community peer review system and a postgraduate-level discussion forum analogous to MathOverflow.

Welcome to PhysicsOverflow! PhysicsOverflow is an open platform for community peer review and graduate-level Physics discussion.

Please help promote PhysicsOverflow ads elsewhere if you like it.

News

PO is now at the Physics Department of Bielefeld University!

New printer friendly PO pages!

Migration to Bielefeld University was successful!

Please vote for this year's PhysicsOverflow ads!

Please do help out in categorising submissions. Submit a paper to PhysicsOverflow!

... see more

Tools for paper authors

Submit paper
Claim Paper Authorship

Tools for SE users

Search User
Reclaim SE Account
Request Account Merger
Nativise imported posts
Claim post (deleted users)
Import SE post

Users whose questions have been imported from Physics Stack Exchange, Theoretical Physics Stack Exchange, or any other Stack Exchange site are kindly requested to reclaim their account and not to register as a new user.

Public \(\beta\) tools

Report a bug with a feature
Request a new functionality
404 page design
Send feedback

Attributions

(propose a free ad)

Site Statistics

205 submissions , 163 unreviewed
5,047 questions , 2,200 unanswered
5,345 answers , 22,709 comments
1,470 users with positive rep
816 active unimported users
More ...

  Question about trivializing an SPT phase via group extension.

+ 5 like - 0 dislike
1580 views

Consider a d (spatial) dimensional SPT phase with an on-site symmetry G, classified by some non-trivial cocycle  $\omega^{d+1}(\{g_i\}) \neq \delta \mu^d(\{g_i\})$, $[\omega^{d+1}(\{g_i\})] \in H^{d+1}(G,U(1))$, . In a recent paper, Wang, Wen and Witten construct gapped boundaries via a suitable group extension $1 \longrightarrow H  \overset{i}{\longrightarrow} K  \overset{r}{\longrightarrow} G  \longrightarrow 1 $ such that the cocycle fro $H$ defined via pullback is trivial $r^*\omega^{d+1}(\{h_i\}) = \omega^{d+1}(\{r(h_i)\})= \delta \mu^d(\{h_i\})$. The gapped boundary corresponds to an $H$ invariant theory but with $K$ gauged so that the global symmetry is $H/K \cong G$ as required. The following sentence they say however confuses me: 

" By definition, two states in two different $G$-SPT phases cannot smoothly deform into each other via deformation paths that preserve the $G$-symmetry. However, two such $G$-SPT states may be able to smoothly deform into each other if we view them as systems with the extended $H$-symmetry and deform them along the paths that preserve the $H$-symmetry. " 

To me, it sounds like the two sentences contradict each other.
Q1) If there is an $H$ invariant deformation path to connect the system to a trivial state, then does that not automatically give us a $G$ invariant deformation path?
Q2) Does sentence 2 somehow only apply to the boundary rather than the bulk? 

asked Sep 14, 2017 in Theoretical Physics by abhishodh (65 points) [ no revision ]

Please give a background reference. What is SPT? 

What is SPT?

I apologize for the delayed response. The response by Xiao-Gang Wen in this post in physics stack exchange gives a nice explanation on what a Symmetry Protected Topological (SPT) phase is and is not. https://physics.stackexchange.com/questions/135398/definition-of-short-range-entanglement

1 Answer

+ 1 like - 0 dislike

The point is indeed a subtle one. If you begin with a G SPT and consider it as a K SPT, then there is still no local unitary commuting with the K action which can take you to a product state, since the generators of K are just the generators of G. However, if you allow the introduction of new ancilla bits to your sites, then it matters whether we consider it a G or a K action since we must extend our operators to act on the ancillas as well. If it's to be a G action, this is a stricter set of relations that the new operators need to satisfy than if it's to be merely a K action.

answered Sep 22, 2017 by Ryan Thorngren (1,925 points) [ no revision ]

Thanks for your answer Ryan. I understand this better after working through a few examples. For the benefit of anyone else who might stumble upon this post, I would also like to add that  within K, the original generators of G no longer form a subgroup. This means that when we add ancillary spins that transform faithfully under the extended group, K, the system is no longer G invariant. Hence, any K-invariant circuit that trivializes the G SPT state is also not G invariant which is consistent with standard SPT assertions. 

Your answer

Please use answers only to (at least partly) answer questions. To comment, discuss, or ask for clarification, leave a comment instead.
To mask links under text, please type your text, highlight it, and click the "link" button. You can then enter your link URL.
Please consult the FAQ for as to how to format your post.
This is the answer box; if you want to write a comment instead, please use the 'add comment' button.
Live preview (may slow down editor)   Preview
Your name to display (optional):
Privacy: Your email address will only be used for sending these notifications.
Anti-spam verification:
If you are a human please identify the position of the character covered by the symbol $\varnothing$ in the following word:
p$\hbar$ysi$\varnothing$sOverflow
Then drag the red bullet below over the corresponding character of our banner. When you drop it there, the bullet changes to green (on slow internet connections after a few seconds).
Please complete the anti-spam verification




user contributions licensed under cc by-sa 3.0 with attribution required

Your rights
...